Shared Flashcard Set

Details

CEN Test Prep Questions
CEN Test Prep Questions
145
Health Care
Professional
04/18/2011

Additional Health Care Flashcards

 


 

Cards

Term

The most common symptom of a patient who has a pinworm infection is

A.

diarrhea.

B.

anorexia.

C.

abdominal pain.

 

D.

perianal pruritus.

Definition

D.

perianal pruritus.

Pinworms are the most common cause of anal pruritis in children.

Emergency Care of Women, Tintinalli, 4th ed., p. 487

Term

A patient is being discharged with a presumptive diagnosis of primary herpes simplex viral infection. The patient understands discharge instructions if she states,

 

A.

"Acyclovir will cure my herpes."

B.

"My boyfriend will not get this if he uses a condom."

 

C.

"If I get pregnant, I can pass the disease to my baby."

D.

"I know that this first episode will last only 4 or 5 days."

Definition

C.

"If I get pregnant, I can pass the disease to my baby."

The greatest risk to the fetus occurs when overt HSV infection is present at the time of delivery. In this situation, the principal mechanism of infection is direct contact with the infected vesicles during the process of vaginal birth.

Emergency Care of Women, Tintinalli, 4th ed., p. 291

Term

A patient comes to the emergency department 30 minutes after an insect bite to the leg. The wound is erythematous, swollen, and indurated. The emergency nurse should first

 

A. perform an Allen's test.

B. examine the inguinal lymph nodes.

C. check the pulses proximal to the wound.

D. evaluate inspiratory and expiratory breath sounds.

Definition

D.

evaluate inspiratory and expiratory breath sounds.

Patients with insect bites are at risk for anaphylactic reactions. In assessing for systemic reactions, ABC are a priority. Assessment of breathing includes evaluating breath sounds.

Emergency Nursing Core Curriculum, 6th ed., p. 333

Term

Which of the following is the most common finding in pulmonary embolism?

 

A. dyspnea

B. crackles (rales)

C. fever

D. hemoptysis

Definition

A. dyspnea

With pulmonary embolus, common signs and symptoms include shortness of breath, tachypnea, tachycardia, and sudden-onset pleuritic chest pain that increases with respirations. Dyspnea is the most common sign of pulmonary embolism. Crackles and hemoptysis are likely to be associated pulmonary edema. Fever is a finding in patients with a pulmonary embolism, but it is not common.

Sheehy's Emergency Nursing, Principles and Practice, 5th ed., p. 442
Core Curriculum for Critical Care Nursing, 6th ed., p. 717-719
Sheehy's Manual of Emergency Care, 6th ed., p. 308-309

Term

Assessment of a patient with severe acute pancreatitis reveals ascites and diminished breath sounds in the right lower lobe. Vital signs are: BP 109/70 mm Hg, HR 110/min, and RR 38/min. The emergency nurse should suspect that the patient has developed

A. pneumonia.

B. a pneumothorax.

C. pleural effusion.

D. a diaphragmatic hernia.

Definition

C.

pleural effusion.

Pleural effusion is an excess of fluid trapped in the intrapleural space of the lungs. In pancreatitis, this occurs because fluid associated with ascites moves from the peritoneal space into the pleural space through lymph channels. Seventy percent of pleural effusions occur on the right side.

Emergency Nursing Core Curriculum, 6th ed., p. 176
Sheehy's Emergency Nursing, Principles and Practice, 5th ed., p. 542-543

 

Term

A confused elderly woman has a blood glucose level of 1175 mg/dL (65 mmol/L). Serum ketones are negative and arterial blood gases show a pH of 7.25. The emergency nurse should anticipate an order to administer which of the following first?

 

A.

sodium bicarbonate

B.

potassium chloride

C.

high-dose insulin

D.

large volumes of IV fluids

Definition

D.

large volumes of IV fluids

Increased glucose, negative ketones, and acidosis are signs and symptoms of hyperosmolar hyperglycemic nonketotic coma. In the treatment of this state, the goals are rehydration and correcting electrolyte imbalances before potassium replacement and insulin.

Emergency Nursing Core Curriculum, 6th ed., p. 299
Mosby's Emergency Nursing Reference, 3rd ed., p. 326

Term

A patient who has been in a motorcycle crash is awake, alert, and oriented, but complains of severe neck pain. A cross-table lateral radiograph demonstrates a fracture of the second and third cervical vertebrae. Shortly after the radiograph is taken, the patient experiences respiratory arrest. Appropriate initial management of the airway should consist of

 

 

A.

endotracheal intubation.

B.

administration of oxygen by a nasal cannula.

C.

jaw-thrust maneuver and ventilation with a bag-mask device.

D.

head-tilt maneuver and placement of an oropharyngeal airway.

Definition

C.

jaw-thrust maneuver and ventilation with a bag-mask device.

The jaw-thrust maneuver should be used as the basic airway management in any patient with a suspected cervical spine injury without tilting the head. Endotracheal intubation will be done, but is not the first action to be taken.

Core Curriculum for Critical Care Nursing, 6th ed., p. 10

Term

Which of the following statements should be included in discharge instructions for a patient who has undergone treatment of epistaxis?

 

A.

"Drink plenty of hot fluids."

B.

"Open your mouth while sneezing."

C.

"Avoid humidity in the environment."

D.

"Reduce sodium intake."

Definition

B.

"Open your mouth while sneezing."

Sneezing with the mouth closed will force air through the nasal passages, possibly dislodging the clot; therefore, starting the bleeding again.

Rosen's Emergency Medicine Concepts and Clinical Practice, 6th Ed., p. 1074
Jean Proehl, Emergency Nursing Procedures, 4th Ed., 2008, p. 814

Term

An adult patient sustains blunt trauma to the abdomen in a motor vehicle crash. The patient refuses to allow any invasive treatments. Initial assessment reveals an ecchymotic area in the left upper quadrant. The emergency nurse should

 

A.

allow the patient to leave after signing a medical release form.

B.

request an order to sedate the patient.

 

C.

assess the patient's neurological status.

D.

obtain consent from a relative for treatment.

Definition

C.

assess the patient's neurological status.

A rational, competent adult can refuse any treatment. The first step is to establish the patient's ability to understand the consequences of refusing treatment. Due to the mechanism of injury, this patient should be evaluated for the presence of a head injury.

Sheehy's Manual of Emergency Care, 6th ed., p. 25
Sheehy's Emergency Nursing, Principles and Practice, 5th ed., p. 17

Term

The best indication of adequate tissue perfusion in a patient in shock is

 

A.

a urine output of 30 mL/h.

B.

an oxygen saturation of greater than 70%.

C.

a central venous pressure of 10 cm H2O.

D.

a capillary refill time of greater than 2 seconds.

Definition

A.

a urine output of 30 mL/h.

Urine output is the best indicator of adequate tissue perfusion. It is desired to have an oxygen saturation of much greater than 70% and a capillary refill time of LESS than 2 seconds.

Sheehy's Emergency Nursing, Principles and Practice, 5th ed., 2003. p. 510
Sheehy's Manual of Emergency Care, 6th ed., p. 360

Term

During quality review, the quality improvement committee identify a variance from the standard of nursing care of two patients with chest pain. Which of the following is the most appropriate recommendation by the committee?

 

A.

Schedule a nursing inservice on the standard of care for chest pain.

B.

Design a new flow sheet for cardiac documentation.

C.

Request the nurse manager to discipline the nurses involved.

D.

Conduct a random audit and determine the frequency of the problem.

Definition

A.

Schedule a nursing inservice on the standard of care for chest pain.

Conducting a random audit is most appropriate in this scenario. The quality improvement committee needs to determine whether these two cases are isolated incidents or if there is an ongoing deficiency in the standard of care delivered. Inservices, new flow sheets, or discipline are not considered until the scope of the problem is defined.

Sheehy's Emergency Nursing, Principle and Practice, 5th ed., p. 53

Term

Which of the following is the most likely early complication of myocardial infarction?

 

A.

ventricular dysrhythmia

B.

ruptured myocardium

C.

elevated blood pressure

D.

congestive heart failure

Definition

A.

ventricular dysrhythmia

Ventricular dysrhythmias are due to the decrease in myocardial perfusion and oxygenation, which causes an increase in irritability of the heart muscle. This is a common complication that usually occurs within 72 hours of myocardial injury.

Emergency Nursing Core Curriculum, 6th ed., p. 67

Term

A patient is being treated for an acute exacerbation of chronic obstructive pulmonary disease. An improvement in the patient's condition is indicated if the patient is

 

A.

coughing up thick, yellow sputum.

B.

maintaining the tripod position.

C.

using pursed-lip breathing techniques.

D.

able to speak in uninterrupted sentences.

Definition

D.

able to speak in uninterrupted sentences.

General assessment of a patient with a respiratory emergency includes speech pattern (i.e., how many words can a patient speak before needing to breathe? How many phrases or full sentences can the patient state?). Purse-lip breathing and tripod position both suggest continued distress. Thick yellow sputum is not unexpected and neither suggest improvement or deterioration.

Sheehy's Manual of Emergency Care, 6th ed., p. 307, 318

Term

Two children die in the emergency department within 8 hours. The staff members who cared for these children are now sitting alone and staring. The charge nurse should

 

A.

arrange time off for affected staff.

B.

schedule a critical incident stress debriefing.

C.

suggest that they consider temporary transfer to another unit.

D.

explain to them that these situations are part of the job.

Definition

B.

schedule a critical incident stress debriefing.

 

Death of children in the emergency care environment can be a stressor to staff. The staff in this scenario are exhibiting signs of stress which can be exhibited through physical, cognitive, emotional and behavioral factors. The consequences of untreated stress may lead to more serious problems for the staff. Critical incident stress debriefing is the recommended practice to assist emergency staff in dealing effectively with stressors.

ENA Position Statement, Stress Management Strategies
ENPC, 3rd ed., p. 267-268

Term

A victim of near-drowning who was initially thought to be "OK" arrives at the emergency department several hours later with increasing shortness of breath. Respirations are 36/min, and crackles (rales) and wheezes are present. The patient receives 100% oxygen via nonrebreathing mask. Arterial blood gas analysis yields the following values:

pH 7.30; PaCO2 40 mm Hg; PaO2 50 mm Hg; and HCO3- 18 mEq/L

The emergency nurse should anticipate the need to first

 

A.

prepare for intubation of the trachea.

B.

decrease the amount of oxygen.

C.

administer a bronchodilator.

D.

administer 50 mEq of sodium bicarbonate.

Definition

A.

prepare for intubation of the trachea.

Submersion injuries frequently progress to respiratory failure (ARDS). Using the ABC method of assessment and the results of the blood gas analysis on 100% oxygen, this patient is not adequately breathing. To better provide support for oxygenation, the emergency nurse should prepare for intubation.

Sheehy's Manual of Emergency Care, 6th ed., p. 325

Term

Which of the following patients should receive the highest priority of care upon presentation to the emergency department?

 

A.

a patient who is brought in by the police who is confused and unkempt.

B.

a patient whose medical history is unremarkable, but complains of flank pain.

C.

a diabetic patient with a lacerated finger.

D.

a patient without previous health problems complains of lower back pain.

Definition

A.

a patient who is brought in by the police who is confused and unkempt.

The presentation of the unkempt patient who was brought in by police describes a situation in which little is known about the patient's status. Since the patient is confused, an altered mental status is evident. Patients with altered mental states take priority over patients without life-threatening airway, breathing, or circulation problems.

Sheehy's Manual of Emergency Care, 6th ed., p. 61-81
Sheehy's Emergency Nursing, Principles and Practice, 5th ed., p. 75-83

Term

Immediate intervention for a patient who has a large dermal avulsion injury of the arm should focus on

 

A.

minimizing scarring.

B.

preventing infection.

C.

controlling hemorrhage.

D.

minimizing the loss of mobility.

Definition

C.

controlling hemorrhage.

Avulsions are characterized by full-thickness tissue loss that prevents wound approximation. Hemostasis is the immediate problem. Immediate intervention should focus on controlling bleeding.

Options A, C, and D are incorrect because controlling hemorrhage needs to occur prior to the ongoing preventive care associated with minimizing scarring, loss of mobility, and preventing infection.

Emergency Nursing Core Curriculum, 6th ed. p. 746

Term

A sickle cell crisis may be precipitated by

 

A.

exposure to heat.

B.

exposure to cold.

C.

an increase in humidity.

D.

a decrease in humidity.

Definition

B.

exposure to cold.

The underlying pathophysiology is sludging of sickled red blood cells in the microcirculation which results in infarction. The event can be precipitated by infection, exposure to cold, dehydration, and high altitude.

Sheehy's Emergency Nursing, Principles and Practice, 5th ed., p. 638
Core Curriculum for Critical Care Nursing, 6th ed., p. 422

Term

Neurogenic shock is most likely indicated if a patient has a blood pressure of

 

A.

  86/50 mm Hg and a pulse of 52/min.

B.

100/80 mm Hg and a pulse of 146/min.

C.

120/74 mm Hg and a pulse of 110/min.

D.

180/98 mm Hg and a pulse of 45/min.

Definition

A.

  86/50 mm Hg and a pulse of 52/min.

Neurogenic shock is a result of injury to the medullary vasomotor center or to the spinal cord. Resultant block of the sympathetic nervous system leaves the parasympathetic nervous system unopposed. Signs and symptoms of neurogenic shock include decreased blood pressure with an associated bradycardia.

Sheehy's Manual of Emergency Care, 5th ed., p. 162-163
Core Curriculum for Critical Care Nursing, 6th ed., p. 732

 

Term

A 3-year-old girl who has acute respiratory distress is admitted to the emergency department. She is sitting forward and drooling, and her neck is extended. These findings support the diagnosis of

 

A.

croup.

B.

epiglottitis.

C.

bronchiolitis.

D.

aspiration of a foreign body.

Definition

B.

epiglottitis.

Epiglottitis is a condition characterized by edema of the epiglottis and the patient is at risk for airway obstruction. The patient will find a position that will maximize air exchange, and drooling indicates that edema is interfering with swallowing.

Core Curriculum for Critical Care Nursing, 6th ed., p. 285-286

 

Term

A patient who has periorbital bruising and cheek flattening after being kicked by a horse has an unsteady gait on arrival at the emergency department. The emergency nurse should first

 

A.

order radiographs of the face.

B.

place the patient in spinal precautions.

C.

palpate the infraorbital rim.

D.

evaluate visual acuity and pupillary response.

Definition

B.

place the patient in spinal precautions.

Initial care of a trauma patient includes considering the mechanisms of injury and following the Primary assessment, which consists of:

A is for airway AND c-spine
B is for breathing
C is the circulation
D is for disability (perform a neurological assessment)
E is for expose and environment

The ED nurse must assume a c-spine injury until proven otherwise.

Radiographs will be done, BUT are not the first step in the care of this patient.
Palpation of the face will be done during the secondary assessment. Visual acuity will be done as a part of the secondary assessment. Pupillary response may be done during "D" in the initial assessment, but this is after c-spine precautions have been initiated.

Sheehy's Manual of Emergency Care, 6th ed., p. 740.

Term

A man who briefly lost consciousness after a motor vehicle crash is alert and oriented upon arrival at the emergency department. The patient states that he is fine except for an abrasion where his head hit the door frame of the automobile. The patient complains of blurred vision 1 hour later. The emergency nurse should first

 

A.

assess visual acuity.

B.

obtain orthostatic vital signs.

C.

repeat the neurologic assessment.

D.

obtain blood glucose measurement.

Definition

C.

repeat the neurologic assessment.

Considering this patient’s mechanism of injury, a closed head injury should be suspected. Increased intracranial pressure due to edema or intracranial bleeding may be manifested by symptoms such as visual disturbances, decreased level of consciousness or vomiting; therefore, a follow-up neurological assessment is warranted.

There is no reason to suspect that this patient would have orthostatic changes in vital signs. Assessing visual acuity and obtaining a blood glucose measurement may be indicated if the patient’s neurological status is unchanged but they would not be the priority.

Mosby’s Emergency Nursing Reference, 3rd ed., p. 525
Sheehy's Manual of Emergency Care, 6th ed., p. 613

Term

Discharge teaching for the caregiver of a patient with a concussion would be considered effective if the caregiver

 

A.

keeps the patient awake all night.

B.

withholds fluids for 12 hours when the patient is nauseated.

C.

seeks assistance when the patient develops lethargy.

D.

calls the emergency department when the patient is unable remember the event.

Definition

C.

seeks assistance when the patient develops lethargy.

A change in the patient's level of consciousness is an early sign of increased intracranial pressure and needs further evaluation and prompt interventions.

A is based on a misconception that a person should not sleep after a head injury because the emergency nurse might not be able to wake them. B is based on a traditional belief that all patients with head injuries will vomit. Although this is a common complaint, withholding fluids for 12 hours is an unnecessary prevention action. D is incorrect as some memory loss is common and does not require immediate interventions.

Trauma Nursing Core Course Manual, 6th ed., p. 97
Emergency Nursing Core Curriculum, 6th ed., p. 829

Term

Anaphylactic reactions to substances administered systemically usually occur

 

A.

within 1 hour.

B.

between 1 and 2 hours.

C.

between 2 and 24 hours.

D.

after 24 hours.

Definition

A.

within 1 hour.

Substances administered systemically reach the circulation rapidly and a reaction will occur quickly.

Sheehy's Emergency Nursing, Principles and Practice, 5th ed., 2003. p. 513

 

Term

Which of the following drugs is commonly used to treat symptomatic bradycardia?

 

A.

atropine sulfate

B.

calcium chloride

C.

metoprolol (Lopressor)

D.

diltiazem (Cardizem)

Definition

A.

atropine sulfate

Atropine sulfate is the drug of choice for the treatment of symptomatic bradycardia. Other therapies for bradycardia include pacing, sympathomimetic agents, and oxygenation (particularly in children). Metoprolol (a beta-blocker) and diltiazem (a calcium channel blocker) are both used to reduce heart rate and cardiac work.

Emergency Nursing Core Curriculum, 6th ed., p. 217

Term

A patient who has a suspected T4 spinal injury sustained in a fall should be monitored for signs of neurogenic shock, including

A. autonomic dysreflexia.

B. diaphragmatic paralysis.

C. increased intracranial pressure.

D. loss of sympathetic function.

Definition

D.

loss of sympathetic function.

In neurogenic shock, loss of sympathetic function below the level of injury and an unopposed parasynthetic influence causes hypotension and bradycardia. Autonomic dysreflexia and diaphragmatic paralysis maybe associated with spinal cord injury, but not neurogenic shock. Spinal cord injury does not produce increased intracranial pressure.

Emergency Nursing Core Curriculum, 6th ed., p. 844

 

Term

When caring for a patient with a suspected blunt cardiac injury, the emergency nurse should first prepare to administer

A. supplemental oxygen.

B. a narcotic analgesic.

C. lidocaine (Xylocaine).

D. sublingual nitroglycerin (Anginine).

Definition

A.

supplemental oxygen.

Blunt cardiac injury, formerly referred to as cardiac contusion or myocardial contusion, is best diagnosed with an echocardiogram and is treated symptomatically. Monitor the patient's cardiac rhythm, administer supplemental oxygen, and treat the patient's musculoskeletal chest pain. Dysrhythmias (premature ventricular complexes) are common, but are rarely treated unless they interfere with hemodynamic stability.

Emergency Nursing Core Curriculum, 6th ed., p. 947-948

 

Term

Which of the following signs or symptoms should lead an emergency nurse to suspect that a woman who is in labor is ready to deliver?

 

A.

The cervix is dilated to 7 cm.

B.

Contractions are 3 minutes apart.

C.

The amniotic membrane has ruptured.

D.

The woman has a desire to bear down.

Definition

D.

The woman has a desire to bear down.

The desire to bear down or "push" is clearly a sign of impending delivery, secondary to the pressure of the descending fetus stimulating the mother's desire to push. Under normal circumstances the cervix needs to dilate to 10 cm in order for the woman to deliver. Contractions at 3 min apart simply suggest active labor and not impending delivery. Amniotic membranes can rupture hours to days before actual delivery.

Emergency Nursing Core Curriculum, 6th ed., p. 352

Term

Orthopnea, diaphoresis, anxiety, moist crackles (rales), and cough are usually associated with

 

A.

pneumonia.

B.

pulmonary embolism.

C.

bronchial asthma.

D.

pulmonary edema.

Definition

D.

pulmonary edema.

In left ventricular failure, the cardiac output declines and pulmonary venous pressure increases. This results in an accumulation of fluid in the pulmonary tissue and alveoli.

Sheehy's Manual of Emergency Care, 5th ed., p. 207-208
Emergency Nursing Core Curriculum, 6th ed., p. 204-205

 

Term

A patient who sustains a severe pulmonary contusion as a result of a motor vehicle crash should be expected to have

 

A.

steady, substernal pain and no radiographic changes.

B.

hypotension and muffled breath sounds.

C.

local areas of wheezing and radiographic changes.

D.

neck vein distention and diminished breath sounds on the affected side.

Definition

C.

local areas of wheezing and radiographic changes.

In a pulmonary contusion, blood extravasates into the lung tissue, causing alveolar and interstitial edema. This may cause localized wheezing and may be evident on radiograph, though usually 12 or more hours after the event.

Emergency Nursing Core Curriculum, 6th ed., p. 941-942

 

Term

An emergency nurse suspects that a patient has hepatitis. Which of the following precautions would be most appropriate for health care personnel?

 

A.

Allow only staff who have received hepatitis immunization to care for the patient.

B.

Wear gloves if there will be any contact with the patient's fluid or secretions.

C.

Keep the patient in a single room with laminar airflow.

D.

Ensure that the patient wears a mask, gown, and gloves if transported to the radiology department.

Definition

B.

Wear gloves if there will be any contact with the patient's fluid or secretions.

Hepatitis is inflammation of the liver that is commonly caused by one of several viruses, which are variously transmitted via either fecal-oral or blood/semen contact.

Emergency Nursing Core Curriculum, 6th ed., p. 451-455.

Term

A patient who has ingested antifreeze containing ethylene glycol is now undergoing treatment. A POOR response to treatment would be indicated by

 

A.

pupillary constriction.

B.

metabolic acidosis.

C.

rotary nystagmus.

D.

hyperreflexia.

Definition

B.

metabolic acidosis.

Ethylene glycol is metabolized to produce glycolic which results in metabolic acidosis.

Core Curriculum for Critical Care Nursing, 6th ed., p. 650

Term

A patient who has an infection returns to the emergency department. After determining that the patient has not been taking the prescribed antibiotics, the nurse reviews the instructions with the patient. Which of the following responses by the nurse is most appropriate?

 

A.

"Tell me how you will take the antibiotics."

B.

"Here is a list of times you will take the antibiotics."

C.

"Do you have any questions about how to take the antibiotics?"

D.

"If you don't take the antibiotics, you will need to return for injections."

Definition

A.

"Tell me how you will take the antibiotics."

To evaluate learning, the nurse should ask the patient to give the instructions back to the nurse and in the patient's own words. This reinforces learning for the patient and identifies where further instructions may be necessary.

Emergency Nursing Core Curriculum, 6th ed., p. 1006

Term

Which of the following findings suggests that a patient may have a ruptured ovarian cyst?

 

A.

intermittent abdominal pain

B.

sharp abdominal pain

C.

bloody vaginal discharge

D.

thick yellow vaginal discharge

Definition

B.

sharp abdominal pain

The pain associated with a ruptured ovarian cyst is characterized as sharp and constant rather than intermittent. There is usually no associated vaginal discharge.

Emergency Nursing Core Curriculum, 6th ed., p. 559

Term

Two weeks after having a streptococcal throat infection, a healthy adult experiences chest pain and is diagnosed as having pericarditis. On auscultation, the emergency nurse should expect to hear

 

A.

a heart murmur.

B.

bilateral wheezes.

C.

a friction rub.

D.

an S4 gallop.

Definition

C.

a friction rub.

Pericarditis, regardless of its etiology, causes inflammation of the pericardial sac. This condition does not affect the heart valves so it does not produce a murmur. Gallops are related to increased fluid volume or heart failure. In the patient with pericarditis, a coarse sound is noted on cardiac auscultation. This sound is produced when inflamed pericardial tissue rubs against the heart with each cardiac contraction.

Emergency Nursing Core Curriculum, 6th ed., p. 212

Term

Which of the following statements about subdural hemorrhage is true?

 

A.

It results from a thrombosed artery.

B.

It results from a tear in the middle meningeal artery.

C.

It may initiate a rapid or slow onset of symptoms.

D.

It occurs between the skull and the dura.

Definition

C.

It may initiate a rapid or slow onset of symptoms.

A subdural hemorrhage is usually due to venous bleeding that results from tearing of bridging veins. An acute subdural hemorrhage has signs and symptoms within 48 hours after injury. A chronic subdural hemorrhage may not have signs or symptoms for 2 weeks or longer.

Trauma Nursing Core Course Manual, 6th ed., p. 100
Emergency Nursing Core Curriculum, 6th ed., p. 837

 

Term

Assessment of a patient who sustains a bite from a black widow spider is likely to reveal

A. diarrhea.

B. hypothermia.

C. hypotension.

D. muscle spasms.

Definition

D.

muscle spasms.

Systemic reactions to black widow spider bites develop within 1 hour and include nausea, vomiting, hypertension, hyperactive deep tendon reflexes, elevated temperature, and muscle spasms. Seizures and shock may also develop.

Sheehy's Emergency Nursing, Principles and Practice, 5th ed., p. 629

Term

A patient who has a neck injury is to be moved. The emergency nurse should protect the patient's cervical spine by

 

A.

stabilizing the head and neck until the cervical collar is applied.

B.

immobilizing the head by placing a hand over the forehead and chin.

C.

asking the patient not to move his head.

D.

placing the patient on a long backboard.

Definition

A.

stabilizing the head and neck until the cervical collar is applied.

When a cervical injury is suspected, the head should be manually maintained in a neutral position while a rigid cervical collar is placed. Simply placing the patient on a backboard or placing the hands over the chin and forehead will not affectively immobilize the cervical spine. Even the most compliant patient cannot prevent spontaneous head movement.

Sheehy's Manual of Emergency Care, 6th ed., p. 646

 

Term

A patient presents with a history of abdominal pain, nausea, and vomiting for 2 days. Upon standing, the patient complains of dizziness. The emergency nurse should anticipate an order to do which of the following first?

 

A.

Initiate IV access.

B.

Administer pain medication.

C.

Insert a nasogastric tube.

D.

Draw blood for a CBC.

Definition

A.

Initiate IV access.

The patient's symptoms suggest postural hypotension which is most likely secondary to hypovolemia. This is treated with IV fluid bolus. Laboratory values can be drawn to evaluate the patient's status, but the priority intervention should be fluid replacement. Pain medications and the nasogastric tube may be necessary, but not until the fluid deficit is addressed.

Core Curriculum for Critical Care Nursing, 6th ed., p. 727
Sheehy's Manual of Emergency Care, 6th ed., p. 362-363

 

Term

A patient who sustains a head injury has increased pressure on the left oculomotor nerve. Assessment of the left eye is most likely to reveal

 

A.

conjunctival edema.

B.

ptosis of the eyelid.

C.

dilation of the pupil.

D.

ciliary spasm of the eyelid.

Definition

C.

dilation of the pupil.

Oculomotor nerve (cranial nerve III) regulates pupil size and reactivity on the side ipsilateral to the injury.

Sheehy's Manual of Emergency Care, 6th ed., p. 615

Term

The initial drug of choice in the treatment of anaphylaxis is

 

A.

epinephrine (Adrenalin).

B.

prednisone (Sterapred).

C.

isoproterenol (Isuprel).

D.

diphenhydramine (Benadryl).

Definition

A.

epinephrine (Adrenalin).

The mechanism of anaphylaxis includes vasodilation that is initially treated with epinephrine. Other medications with other mechanisms of action are used, but do not take priority.

Sheehy's Emergency Nursing, Principles and Practice, 5th ed., 2003. p. 513
Emergency Nursing Core Curriculum, 6th ed., p. 502

 

Term

Bacterial meningitis is most strongly suggested by a fever and

A. confusion.

B. slurred speech.

C. nuchal rigidity.

D. lateral nystagmus.

Definition

C.

nuchal rigidity.

The classic triad of fever, stiff neck, and altered level of consciousness is noted in approximately 2/3 of adult patients with meningitis.

Sheehy's Emergency Nursing, Principles and Practice, 5th ed., 2003, p. 525
Emergency Nursing Core Curriculum, 6th ed., p. 460

 

Term

A patient who has a long-leg cast has pain on passive movement and decreased sensation in the affected leg. Urinalysis reveals myoglobinemia. These findings most likely indicate

 

A.

fat emboli.

B.

nerve laceration.

C.

compartment syndrome.

D.

renal artery occlusion.

Definition

C.

compartment syndrome.

A characteristic sign of compartment syndrome is pain with passive movement of the involved extremity. Muscle damage due to compression/damage of the muscle releases myoglobin, which will be evident in the urinalysis.

Trauma Nursing Core Course Manual, 6th ed., p. 188-189
Emergency Nursing Core Curriculum, 6th ed., p. 922-923

Term

Ongoing evaluation of a patient with blunt liver trauma includes serial measurements of which of the following laboratory values?

 

A.

hematocrit

B.

lipase

C.

troponin

D.

amylase

Definition

A.

hematocrit

As the patient can continue to bleed internally from blunt liver trauma without overt external evidence, the ongoing laboratory value that would demonstrate this potentially life-threatening problem is the hematocrit. Lipase and amylase are measures for pancreatic injuries. Troponin is a measure for cardiac injury.

Emergency Nursing Core Curriculum, 6th ed., p. 50

Term

The first intervention for an infant with complete airway obstruction after inhalation of a foreign body is

 

A.

intubation.

B.

back blows.

C.

a finger sweep.

D.

abdominal thrusts.

Definition

B.

back blows.

The American Heart Association recommends back blows followed by chest compressions to dislodge an obstruction. Abdominal thrusts should never be performed on an infant nor should a blind finger sweep be used. Intubation takes time and should not be the first intervention.

American Heart Association Emergency Cardiac Care Guidelines, 2005

 

Term

The emergency nurse telephones the mother of a 5-year-old girl who was diagnosed with acute otitis media the previous evening to follow up on the child's care. A 10-day regimen of amoxicillin (Amoxil) had been prescribed. Which of the following statements by the mother indicates the child's care plan needs to be altered?

 

A.

"She has some hives on her chest."

B.

"Her bowel movements have become soft."

C.

"She still hears a little ringing in her ears."

D.

"She still has pain in her ear."

Definition

A.

"She has some hives on her chest."

An allergic reaction to the prescribed amoxicillin (Amoxil) may have occurred and a change in the prescribed antibiotic may be necessary. It is not uncommon for patients to have soft or loose stools with antibiotics. It has only been 24 hours since the patient was seen in the ED. The presence of some continued pain is expected.

Emergency Nursing Core Curriculum, 6th ed., p. 499.

Term

A neonate has just been delivered in the emergency department. After drying, warming, and suctioning, additional intervention is necessary if the neonate has

 

A.

dusky hands.

B.

respirations of 40/min.

C.

a heart rate of 80 beats/min.

D.

a skin temperature of 98º F (36.7º C).

Definition

C.

a heart rate of 80 beats/min.

Neonatal Resuscitation standards state that if the infant's heart rate is less than 100 bpm, respiratory interventions are necessary; if less than 60, then cardiac compressions must be started. Dusky extremities (acrocyanosis) is a normal state for the neonatal period. Respiration of 40 to 60/min is normal for the neonate. Skin temperature of 98° F is normal for the neonate.

ENPC, 3rd ed., p. 181

 

Term

When assessing a wound for a foreign body, the emergency nurse should first

 

A.

soak the wound prior to inspection.

B.

probe the wound manually.

C.

determine the mechanism of injury.

D.

obtain a wound culture.

Definition

C.

determine the mechanism of injury.

The mechanism of injury assists the emergency nurse in assessing the likelihood of a retained foreign body. Foreign bodies in wounds increase the risk of infection and complications.

Sheehy's Manual of Emergency Care, 6th ed., p. 206-207

Term

Results of laboratory studies for a patient who has disseminated intravascular coagulation are most likely to reveal

 

A.

decreased clotting time.

B.

normal fibrinogen levels.

C.

elevation of fibrin split products.

D.

decreased partial thromboplastin time.

Definition

C.

elevation of fibrin split products.

Fibrinogen levels are decreased due to excessive clotting in the vasculature. Clotting times are increased as clotting factors are used up by excessive clotting. As fibrin is broken down (fibrinolysis), the elevated level of circulating fibrin split products further interferes with platelet function.

Emergency Nursing Core Curriculum, 6th ed., p. 415-416
Sheehy's Emergency Nursing, Principles and Practice, 5th ed., p. 639

 

Term

Myocardial infarction is most likely to be fatal

 

A.

during the first 2 hours after the onset of the chest pain.

B.

6 to 12 hours after the onset of the chest pain.

C.

2 to 5 days after the onset of the chest pain.

D.

1 to 2 weeks after the onset of the chest pain.

Definition

A.

during the first 2 hours after the onset of the chest pain.

The most common cause of death following myocardial infarction is a fatal dysrhythmia, ventricular fibrillation, or ventricular tachycardia. These rhythms tend to occur shortly after coronary vessel occlusion as a result of irritation to the myocardium and the heart's electrical conduction system. Death hours or days post-MI is generally the result of cardiogenic shock or reinfarction. Cardiac deaths that occur weeks or months following an MI are largely the result of heart failure.

Sheehy's Emergency Nursing, Principles and Practice, 5th ed., p. 458
Critical Care Challenges: Disorders, Treatments, and Procedures, 2003, p. 2

Term

A patient who has chest pain and palpitations is given a dose of adenosine (Adenocard) by rapid IV push. Intervention has been effective if

 

A.

the chest pain decreases.

B.

systolic blood pressure increases.

C.

there is a transient decrease in heart rate.

D.

supraventricular tachycardia resolves.

Definition

D.

supraventricular tachycardia resolves.

A decrease in chest pain and an improvement in systolic blood pressure are benefits that can be derived from adenosine therapy in the patient with supraventricular tachycardia (SVT). However, resolution of the tachydysrhythmia is the desired endpoint and indicates drug effectiveness. A transient decrease in heart rate following adenosine administration,even frank asystole, indicates the drug has reached the heart, but this is not the endpoint of therapy.

Mosby's Emergency Nursing Reference, IV Medications, 2007, 3rd ed., p. 20-21

Term

Which of the following medications should an emergency nurse anticipate administering to an adult patient who is in status epilepticus?

 

A.

diazepam (Valium)

B.

verapamil (Isoptin)

C.

haloperidol (Haldol)

D.

phenobarbital (Luminal)

Definition

A.

diazepam (Valium)

Diazepam (Valium) is a benzodiazepine used as a first line drug in the treatment of status epilepticus. Verapamil is a calcium channel blocker used to treat rapid heart beat (A-fib, A-flutter, PSVT). Haloperidol is used to treat psychotic disorders and agitation/combative behavior. Phenobarbital is a long-term treatment to prevent seizure-recurrence and is not a first-line medication for status epilepticus.

Sheehy's Manual of Emergency Care, 6th ed., p. 381

 

Term

Hypovolemic shock is most likely to occur secondary to a fracture of the

 

A. humerus.

B. femur.

C. sternum.

D. tibia.

Definition

B.

femur.

A patient may lose up to 1500 mL of blood in a femur fracture. This blood loss correlates with Class II - hemorrhagic shock.

Trauma Nursing Core Course Manual, 6th ed., p. 186-187

Term

During the last trimester of pregnancy, painless vaginal bleeding is most likely indicative of

 

A.

abortion.

B.

uterine rupture.

C.

placenta previa.

D.

abruptio placentae.

Definition

C.

placenta previa.

 

Placenta previa is a condition in which the placenta is abnormally implanted in the lower uterine segment and partially covers the cervical os. Effacement of the cervix in preparation for labor tears the implanted placenta, resulting in painless, bright-red bleeding.

Core Curriculum for Critical Care Nursing, 6th ed., p. 543

Term

A patient who receives dialysis treatments is weak and has muscle cramps. ECG readings show first-degree AV block with peaked T waves. The patient has a heart rate of 70/min and a serum potassium level of 8.0 mEq/L (8.0 mmol/L). The emergency nurse should anticipate administering

 

A.

atropine 0.5 mg, IV push.

B.

lidocaine (Xylocaine) 75 mg, slow IV push.

C.

digoxin (Lanoxin) 0.25 mg, slow IV push.

D.

calcium gluconate 10 mL, slow IV push.

Definition

D.

calcium gluconate 10 mL, slow IV push.

 

Dialysis patients are at risk for fluid and electrolyte disorders. Signs and symptoms of hyperkalemia include peaked T waves and first-degree AV block. Treatment includes 10% calcium gluconate to counter the toxic effects of hyperkalemia on the heart.

Sheehy's Emergency Nursing, Principles and Practice, 5th ed., p. 579
Core Curriculum for Critical Care Nursing, 6th ed., p. 588

 

Term

The skin of a patient in early septic shock is most likely to be

 

A.

cool and pale.

B.

cool and cyanotic.

C.

warm and flushed.

D.

warm and pale.

Definition

C.

warm and flushed.

Signs and symptoms of early septic shock include increased heart rate, increased respiratory rate, and warm, flushed skin, hence the term, "warm shock."

Sheehy's Emergency Nursing Principles and Practice, 5th ed., p. 512
Emergency Nursing Core Curriculum, 6th ed., p. 732-734

 

Term

In which of the following cases has the nurse breached patient confidentiality?

 

A.

notifying the health department about measles

B.

reporting sudden unexpected deaths to the coroner

C.

notifying the police about a possible homicide

D.

reporting HIV status to the patient's employer

Definition

D.

reporting HIV status to the patient's employer

HIV status of a patient is confidential patient information and may not be provided to the patient's employer. The other three options are mandated reportable occurrences the emergency department nurse may encounter.

Sheehy's Manual of Emergency Care, 6th ed., p. 22-23

 

Term

A trauma patient is in the emergency department with suspected cardiac tamponade. The emergency nurse should prepare to treat this condition with

 

A.

pericardiocentesis.

B.

needle thoracostomy.

C.

antidysrhythmics.

D.

transthoracic pacing.

Definition

A.

pericardiocentesis.

Pericardiocentesis involves placing a drain (needle or catheter) through the chest wall and into the pericardial space in order to drain accumulated fluid. In a trauma patient, the involved fluid is blood. Blood can accumulate rapidly in the pericardial sac, a fairly non-distensible structure, causing the ventricles to both fill and empty poorly. This produces an acute decrease in cardiac output.

Needle thoracostomy is indicated for the relief of tension pneumothorax (not a simple pneumothorax). Antidysrhythmic drugs treat dysrhythmias, and transthoracic pacing is for patients with inadequate conduction system activity.

Emergency Nursing Core Curriculum, 6th ed., p. 950

Term

A nurse has difficulty inserting a urinary drainage catheter into a patient with a crushing pelvic injury. This most likely indicates that the

 

A.

bladder is ruptured.

B.

catheter is too large.

C.

catheter is not sufficiently lubricated.

D.

urethra has been injured.

Definition

D.

urethra has been injured.

Urethral injury should be suspected in any patient with pelvic trauma, especially since the symptoms may vary. If resistance to insertion of a urinary drainage catheter is encountered, the procedure should be stopped so as not to cause further damage.

Sheehy's Principles and Practice of Emergency Nursing, 5th ed., p. 311
Emergency Nursing Core Curriculum, 6th ed., p. 800

Term

After a patient has an episode of violent behavior in the waiting room, an important intervention is to

 

A.

allow the patient to go home as soon as possible.

B.

encourage the patient to verbalize his feelings about the situation.

C.

instruct the patient to be quiet and speak only when spoken to.

D.

tell the patient to get himself under control.

Definition

B.

encourage the patient to verbalize his feelings about the situation.

A primary intervention for patients with violent behavior is to "encourage patient to identify feelings."

Emergency Nursing Core Curriculum, 6th ed., p. 682
Sheehy's Manual of Emergency Care, 6th ed., p. 881-882

Term

A patient who sustained major trauma has a blood pressure of 84/56 mm Hg, a pulse of 124/min, and respirations of 42/min. Breath sounds are absent on the left side, the trachea is deviated to the right, and the left thigh is deformed. The emergency nurse should first prepare to assist with

 

A.

application of a traction splint.

B.

needle decompression.

C.

pericardiocentesis.

D.

chest tube insertion.

Definition

B.

needle decompression.

Following the ABC of assessment, the priority for this particular patient is breathing. This patient's respiratory status is compromised due to a tension pneumothorax for which immediate needle decompression is indicated.

Trauma Nursing Core Course Manual, 6th ed., p. 142

Term

After an occlusive dressing has been applied to a patient's open chest wound, the emergency nurse should assess for

 

A.

dyspnea and chest pain.

B.

stridor and hypotension.

C.

dullness on the injured side.

D.

tracheal deviation toward the injured side.

Definition

A.

dyspnea and chest pain.

 

An open chest wound can quickly become a tension pneumothorax if pressure builds inside the chest after an occlusive dressing has been placed. The patient will most likely exhibit increasing pain and dyspnea. The trachea will deviate AWAY from the affected side and the injured side will have hyperresonance.

Core Curriculum for Critical Care Nursing, 6th ed., p. 938

 

Term

A patient with acute cocaine intoxication would be expected to have

 

A.

drowsiness.

B.

flaccid extremities.

C.

first-degree AV block.

D.

rising core body temperature.

Definition

D.

rising core body temperature.

Signs and symptoms of acute cocaine intoxication include tachycardia, hypertension, hyperreflexia, anxiety, restlessness, and elevated body temperature.

Sheehy's Emergency Nursing, Principles and Practice, 5th ed., p. 796
Emergency Nursing Core Curriculum, 6th ed., p. 618

 

Term

An adolescent patient presents to the emergency department with complaints of sore throat, fatigue, malaise, and fever. The patient has cervical lymphadenopathy and pain across the upper abdomen. Which of the following is the best test to help make the diagnosis?

 

A.

Monospot

B.

indirect laryngoscopy

C.

purified protein derivative (PPD) testing

D.

posterior-anterior and lateral chest films

Definition

A.

Monospot

Signs and symptoms of infectious mononucleosis include fatigue, fever, sore throat, cervical lymphadenopathy, splenomegaly and hepatomegaly. The Monospot is the diagnostic lab test used in the process of diagnosing mononucleosis.

The symptoms described are classic to mononucleosis. The Monospot test is the best choice based on the symptoms described. An indirect laryngoscopy is not indicated. The symptoms are not an indication for a PPD test or chest films.

Core Curriculum for Critical Care Nursing, 6th ed., p. 464
Sheehy's Manual of Emergency Care, 6th ed., p. 556-557

Term

A patient experiencing a manic episode of bipolar disorder is likely to have

 

A.

inflated self-esteem.

B.

psychomotor retardation.

C.

complaints of exhaustion.

D.

delusions of persecution.

Definition

A.

inflated self-esteem.

In a manic state, patients exhibit escalated characteristics of personality such as euphoria, arrogance, and inflated self-esteem.

Emergency Nursing Core Curriculum, 6th ed., p. 677

 

Term

Manual uterine massage is recommended for early postpartum hemorrhage because massage will

 

A.

expel retained products of conception.

B.

stimulate uterine contractions.

C.

improve bladder tone.

D.

enhance uterine atony.

Definition

B.

stimulate uterine contractions.

The postpartum uterus may be atonic. Manual stimulation of the uterus results in the muscles and vessels of the uterus contracting, which then reduces intrauterine bleeding.

Sheehy's Manual of Emergency Care, 6th ed., p. 811, 816
Merck Manual

 

Term

A 4-year-old patient is brought to the emergency department the day after having a tonsillectomy. The patient is diaphoretic, tachycardic, and crying, and has obvious oral bleeding. The emergency nurse should first

 

A.

initiate an IV of lactated Ringer's solution at 50 mL/h.

B.

apply pressure to the nose to control bleeding.

C.

administer oxygen via nonrebreathing mask.

D.

suction bloody secretions from the mouth.

Definition

D.

suction bloody secretions from the mouth.

This child's airway is at risk. Following the ABC of assessment, the patient's airway needs to be cleared of blood through suctioning.

Emergency Nursing Core Curriculum, 6th ed., p. 10

Term

Appropriate nursing care for a child who has gastroenteritis includes instructing the parents to

 

A.

withhold dairy products.

B.

give orange juice for rehydration.

C.

offer plain mashed potatoes for solid food.

D.

keep the child NPO for 12 hours.

Definition

A.

withhold dairy products.

Dairy products are a good source of nutrients for invading pathogens. Therefore, they tend to aggravate gastroenteritis. Full strength orange juice contains a lot of sugar, which also aggravates diarrheal illnesses. Plain mashed potatoes are appropriate after the acute phase. Oral fluids should never be withheld from a child with gastroenteritis unless the patient is receiving an adequate amount of intravenous fluids.

Sheehy's Emergency Nursing, Principles and Practice, 5th ed., p. 548

 

Term

A patient with Guillain-Barré syndrome should be expected to have a tingling sensation in the extremities, ascension of symmetric paralysis, and

 

A.

a positive Babinski's sign.

B.

spasticity.

C.

tonic-clonic seizures.

D.

decreased deep-tendon reflexes.

Definition

D.

decreased deep-tendon reflexes.

Signs and symptoms are tingling sensation in the extremities lasting for hours to weeks, severely decreased deep tendon reflexes, and a symmetric paralysis that begins in the lower extremities and ascends. This classic pattern is seen in 90-95% of patients diagnosed with Guillain-Barré syndrome.

Sheehy's Emergency Nursing, Principles and Practice, 5th ed., 2003, p. 528
Emergency Nursing Core Curriculum, 6th ed., p. 533
Sheehy's Manual of Emergency Care, 6th ed., p. 386

 

Term

Chronic arterial insufficiency of the lower extremities is characterized by

 

A.

pitting edema.

B.

bounding pedal pulses.

C.

dilated tortuous veins.

D.

intermittent claudication.

Definition

D.

intermittent claudication.

Claudication is caused by poor tissue perfusion or ischemia due to gradual enlargement of atheromatous plaques.

Core Curriculum for Critical Care Nursing, 6th ed., p. 358
Merck Manual
Emergency Nursing Core Curriculum, 6th ed., p. 245

 

Term

Polystyrene sulfonate resin (Kayexalate) is ordered for a patient who has a serum potassium level of 8.6 mEq/L because the drug

 

A.

forces potassium into the cells.

B.

exchanges sodium for potassium ions.

C.

binds potassium to albumin.

D.

increases renal excretion of potassium.

Definition

B.

exchanges sodium for potassium ions.

Kayexalate is used in the treatment of hyperkalemia because it is a cation exchange resin. Given orally or rectally, the resin is combined with potassium in the colon. The potassium is eliminated in the body through the feces.

Core Curriculum for Critical Care Nursing, 6th ed., p. 377
Clinical Drug Therapy, Rationales for Nursing Practice, 8th ed., p. 925

 

Term

A nebulized bronchodilator is administered to a 3-year-old child with severe asthma. Treatment may be deemed effective if the patient has

 

A.

a respiratory rate that has decreased to 14/min.

B.

a prolonged expiratory phase.

C.

diminished breath sounds bilaterally.

D.

an increased peak expiratory flow rate.

Definition

D.

an increased peak expiratory flow rate.

A diagnostic aid for asthma is serial peak expiratory flow rate measurements before and after each intervention. The hallmark of asthma diagnosis is spirometry before and after bronchodilator therapy to document reversibility of airway narrowing, usually by peak expiratory flow rates.

Sheehy's Manual of Emergency Care, 6th ed., p. 843
Sheehy's Emergency Nursing, Principles and Practice, 5th ed., p. 431

Term

Which of the following factors has the greatest impact on a patient's understanding of discharge instructions?

 

A.

ability and willingness to learn

B.

educational level

C.

learning style

D.

support system

Definition

A.

ability and willingness to learn

Patients with any level of education, learning style, or support system will be less receptive to new information if they are distracted, and teaching will be ineffective. Immediate concerns of the patient will need to be resolved if attention is to be paid to discharge instructions.

Core Curriculum for Critical Care Nursing, 6th ed., 1004

Term

An adult patient who has a small bowel obstruction is awaiting an inpatient bed. Reassessment 4 hours later reveals an increased heart rate, hypotension, and urine output of 15 mL/h. These signs and symptoms are most indicative of which of the following?

 

A.

obstruction of the indwelling urinary catheter

B.

hypovolemia secondary to fluid loss

C.

a ruptured appendix

D.

acute tubular necrosis

Definition

B.

hypovolemia secondary to fluid loss

Tachycardia, hypotension and decreased urine output are classic signs associated with hypovolemia.

Core Curriculum for Critical Care Nursing, 6th ed., p. 167

Term

A patient's attorney asks the emergency nurse questions about the patient's treatment. The nurse should

 

A.

answer all such questions.

B.

provide a copy of the patient's medical record.

C.

refer the attorney to the patient's physician.

D.

refer the attorney to a hospital administrator.

Definition

D.

refer the attorney to a hospital administrator.

The attorney should always be referred to the hospital's administration department. Nurses may only discuss personal information with the patient or their authorized representative.

Emergency Nursing Core Curriculum, 6th ed., p. 1041

 

Term

All of the following factors contribute to infectious complications in oncology patients EXCEPT

 

A.

bone marrow suppression.

B.

altered skin and mucous membrane integrity.

C.

malnutrition.

D.

frequent blood transfusions.

Definition

D.

frequent blood transfusions.

Bone marrow suppression, altered skin and mucous membrane integrity, and malnutrition are all conditions that place an oncology patient more at risk for infection because each of these contributes to compromised immunity. Blood transfusions should assist the patient in building immunity and would not be a factor that contributed to further compromising the patient.

Jean Proehl, Emergency Nursing Procedures, 4th ed., 2008

 

Term

A patient who has a subarachnoid hemorrhage is most likely to describe the headache as

 

A.

throbbing and intermittent.

B.

the "worst headache ever."

C.

mild and persistent.

D.

a dull ache.

Definition

B.

the "worst headache ever."

When an intracranial aneurysm ruptures, blood is forcibly expelled into the subarachnoid space and coats the brain surface. Blood in the subarachnoid space causes obstruction of cerebral blood fluid (CSF) to flow through ventricles. Arterial spasms also occur in a significant number of patients.

Essentials of Critical Care Nursing, 2006, p. 477

 

Term

The most beneficial action of morphine in the treatment of pulmonary edema is to

 

A.

decrease preload.

B.

decrease cardiac reserve.

C.

increase systemic vascular resistance.

D.

increase oxygen consumption.

Definition

A.

decrease preload.

Morphine sulfate has several beneficial effects in the patient experiencing pulmonary edema, particularly pulmonary edema due to heart failure. Morphine reduces pain and anxiety but, most importantly, by increasing venous capacitance (through vasodilation), less blood is delivered to the lungs.

Emergency Nursing Core Curriculum, 6th ed., p. 206

Term

An 11-year-old patient is admitted to the emergency department 1 hour after receiving a blow to the right eye. The patient is unable to look upward with the injured eye. The emergency nurse should suspect

 

A.

hyphema.

B.

vitreous hemorrhage.

C.

blow-out fracture.

D.

traumatic infarction of the optic nerve.

Definition

C.

blow-out fracture.

These symptoms are caused by the ocular motor nerve being trapped within the fractured area of the orbit.

Sheehy's Manual of Emergency Care, 6th ed., p. 745

 

Term

A patient with a history of poor peripheral circulation presents to the emergency department complaining of an achy pain in an entire leg. Which of the following would be the most useful information to obtain?

 

A.

pulse quality in the affected leg

B.

history of a recent leg injury

C.

blood pressure in the affected leg

D.

the patient's usual medications

Definition

A.

pulse quality in the affected leg

Although each of the assessment or exam findings listed are appropriate for the care of this patient, checking pulse quality will most quickly determine whether the patient has a limb threatening emergency. If pulses are normal, the extremity is not immediately at risk.

Emergency Nursing Core Curriculum, 6th ed., p. 245

 

Term

The cardinal signs of facial fractures are pain, swelling, and

 

A.

bleeding.

B.

salivation.

C.

asymmetry.

D.

fractured teeth.

Definition

C.

asymmetry.

Asymmetry is the first facial feature to be assessed after the primary assessment and necessary lifesaving interventions have been completed.

Bleeding can occur from injuries even when a fracture is NOT present. Salivation occurs naturally, and inability to handle secretions is not alone a sign of a facial fracture. Fractured teeth can occur without facial fractures.

Emergency Nursing Core Curriculum, 6th ed., p. 871

Term

Which of the following assessment findings are associated with a skull fracture and would indicate the need for further intervention?

 

A.

rhinorrhea and headache or dizziness

B.

decreasing level of consciousness and restlessness

C.

early evidence of periorbital ecchymosis and photophobia

D.

otorrhea, nausea, and vomiting

 

Definition

B.

decreasing level of consciousness and restlessness

A change in the LOC is an early sign of increased ICP.

Core Curriculum for Critical Care Nursing, 6th ed., p. 830

 

Term

Appropriate treatment for a patient who has frostbite of the lower extremities consists of

 

A.

gentle massage of the affected parts.

B.

rapid rewarming of the affected parts, using hot packs.

C.

immersion of the affected parts in tepid water.

D.

wrapping the extremities in sterile, warm, wet dressings.

Definition

C.

immersion of the affected parts in tepid water.

 

Treatment of frostbite includes warm soaks -- 104 to 110° F. Do not rub, but elevate the extremity.

Sheehy's Manual of Emergency Care, 6th ed., p. 500

 

Term

Which of the following effects of heroin would require the most immediate intervention?

 

A.

altered level of consciousness

B.

respiratory depression

C.

marked decrease in gastric motility

D.

extensive peripheral vasodilation and hypotension

Definition

B.

respiratory depression

 

Patients with significant opioid overdose merit close observation and supportive care for 24 to 48 hours. Respiratory depression can quickly lead to respiratory arrest; therefore, the most immediate response by the nurse is required. An altered level of consciousness does not always place the airway in jeopardy.

Emergency Nursing Core Curriculum, 6th ed., p. 616

 

Term

An essential requirement that needs to be proved in a negligence case is the presence of

 

A.

incorrect action.

B.

failure to act.

C.

damage.

D.

intent to damage.

Definition

C.

damage.

Four elements of negligence that must be proven include duty, breach of duty, proximate cause, and damages. Incorrect actions or failure to act alone are not an essential element of negligence. Every negligence case must prove damages.

Sheehy's Emergency Nursing, Principles and Practice, 5th ed., p. 19-20
Emergency Nursing Core Curriculum, 6th ed., p. 1035

Term

Which of the following signs is an early indication that the condition of a patient with a ruptured diaphragm is deteriorating?

 

A.

Oxygen saturation declines.

B.

QRS widens by 50%.

C.

Bowel sounds are heard in the chest.

D.

Pulse pressure narrows.

Definition

A.

Oxygen saturation declines.

In diaphragmatic rupture, abdominal contents spill into the thoracic cavity, causing respiratory compromise secondary to lung compression. Oxygen saturation decreases as a result. Bowel sounds are heard in the chest would be expected and pulse pressure narrowing is a late sign.

Sheehy's Emergency Nursing, Principles and Practice, 5th ed., p. 302
Trauma Nursing Core Course Manual, 6th ed., p. 139
Sheehy's Manual of Emergency Care, 6th ed., p. 676

 

Term

A 21-year-old patient is suspected of having a left-sided spontaneous pneumothorax. The physical finding that most strongly supports this diagnosis is

 

 

A.

hyperresonance on the left.

B.

increased breath sounds on the left.

C.

dullness on percussion of the right.

D.

an absence of tactile fremitus on the right.

Definition

A.

hyperresonance on the left.

With a pneumothorax, air escapes into the pleural space. Auscultating over this area of trapped air will result in hearing a hyperresonant sound.

Emergency Nursing Core Curriculum, 6th ed., p. 709

 

Term

In planning emergency department care for an infant in shock, the emergency nurse should recognize that

 

A.

rapid infusion of 7.5% sodium bicarbonate is essential to combat acidosis.

B.

moderate volume loss may cause profound hypoperfusion.

C.

hypothermia improves the infant's chance of survival.

D.

tachypnea is the cardinal sign of impending shock.

Definition

B.

moderate volume loss may cause profound hypoperfusion.

Blood volume in a child is 80 mL/kg, less than that of an adult and with a lower oxygen reserve. Volume loss can produce hypovolemia and hypoperfusion more quickly than in adults. Since stroke volume in the pediatric patient is relatively fixed, the child naturally compensates for a decreased cardiac output with tachycardia. Restoration of circulating volume with warmed fluids, blood, and blood products is the treatment of choice.

Sheehy's Principles and Practice, 5th ed., p. 388
Sheehy's Manual of Emergency Care, 6th ed., p. 781

 

Term

Which of the following is most characteristic of early left ventricular failure?

 

A.

peripheral edema

B.

jugular venous distention

C.

expiratory wheezing

D.

paroxysmal nocturnal dyspnea

Definition

D.

paroxysmal nocturnal dyspnea

Left ventricular failure causes blood to back up into the lungs. This can produce paroxysmal nocturnal dyspnea. The other findings listed are symptoms of right heart failure. In LATE left ventricular failure, the right heart will also fail and will then produce peripheral edema, jugular vein distention, and expiratory wheezing ("cardiac asthma").

Emergency Nursing Core Curriculum, 6th ed., p. 204
Core Curriculum for Critical Care Nursing, 6th ed., p. 275

Term

Emergency staff are unable to resuscitate an infant in cardiac arrest. Which of the following findings supports the diagnosis of SIDS?

 

A.

a recent upper respiratory infection

B.

aspiration of a foreign body

C.

old injuries in various stages of healing

D.

a history of frequent otitis media infections

Definition

A.

a recent upper respiratory infection

While many theories have been posited about the cause of SIDS, no consensus exists. Theories include infections, among others.

Sheehy's Manual of Emergency Care, 6th ed., p. 832

Term

Which of the following triage assessments in a toddler would indicate the need for immediate treatment?

 

A.

temperature of 102ºF (38.8ºC)

B.

sinus arrhythmia of 160/min

C.

respirations of 60/min

D.

systolic blood pressure of 90 mm Hg

 

Definition

C.

respirations of 60/min

A respiratory rate of 60/min is a triage red flag.

Children have a decreased pulmonary reserve. A child breathing this fast for any extended period of time without intervention with decompensate very quickly into respiratory failure.

Core Curriculum for Critical Care Nursing, 6th ed., p. 40

 

Term

A patient presents to the emergency department complaining of right elbow pain and swelling following repetitive slot machine use. The most appropriate nursing intervention includes

 

A.

preparation for incision and drainage.

B.

administration of ordered steroids.

C.

administration of ordered antibiotics.

D.

application of a compression dressing.

Definition

D.

application of a compression dressing.

Repetitive use of an extremity should lead the nurse to suspect tendinitis. Tendinitis is the inflammation of tendons and tendon muscle attachment. The appropriate intervention for tendinitis is to immobilize and support the affected area.

Unlike the patient with bursitis, the patient with tendonitis has no fluid collection to drain.

Emergency Nursing Core Curriculum, 6th ed. pg. 595-596
Sheehy's Manual of Emergency Care, 6th ed., pg. 730

 

Term

Which of the following solutions is appropriate for initial resuscitation of patients who have major burns?

 

A.

5% dextrose

B.

lactated Ringer's

C.

0.33% normal saline

D.

5% dextrose in 0.45% normal saline

Definition

B.

lactated Ringer's

Replacement of intravascular volume is the priority intervention for the patient with a thermal injury. Isotonic crystalloid solutions, such as lactated Ringer's or 0.9% normal saline, are recommended for fluid resuscitation.

Sheehy's Manual of Emergency Care, 6th ed., p. 769
Trauma Nursing Core Course Manual, 6th ed., p. 214
Core Curriculum for Critical Care Nursing, 6th ed., p. 812

 

Term

A patient who sustained a gunshot wound is bleeding from the arm. After applying a pressure dressing, the emergency nurse should next evaluate

 

A.

changes in neurovascular status.

B.

development of fever.

C.

presence of bleeding at the dressing.

D.

presence of pain.

Definition

A.

changes in neurovascular status.

A gunshot wound is a missile injury. Bullet wounds may cause neurovascular, bony and soft tissue injuries remote from the projectile path. There is a potential for occult neurovascular injury, especially in high-velocity injuries to the arm. After the intervention of applying a pressure dressing, signs of circulation should be reassessed. Alterations in the neurovascular status indicates further complications.

Fever would generally not be seen immediately following an injury upon arrival in the ED. Unless there is indication of "hemorrhage," a normal amount of bleeding is expected following the initial application of a pressure dressing. Reassessment of any interventions associated with airway, breathing, circulation, and treating underlying conditions related to disease or injury determines the priorities of care.

Emergency Nursing Core Curriculum, 6th ed. p. 751

Term

Which of the following indicates significant dehydration in a 6-month-old infant with a history of vomiting and diarrhea?

 

A.

does not urinate for more than 8 hours

B.

has two bouts of projectile vomiting

C.

has three loose stools in 3 hours

D.

cries loudly during the physical examination

Definition

A.

does not urinate for more than 8 hours

Normal urinary output in a well-hydrated child is 1 mL/kg/h. Decreased urine output is a sign for the need to be evaluated in an emergency department setting.

Emergency Nursing Core Curriculum, 6th ed., p. 367-368.

Term

A patient who has a small puncture wound of the finger from a high-pressure paint gun has swelling in the hand. The emergency nurse should first

 

A.

soak the finger in warm, sterile saline solution.

B.

place the finger on ice.

C.

prepare to obtain a radiograph of the hand.

D.

prepare to administer local anesthetic infiltration.

Definition

C.

prepare to obtain a radiograph of the hand.

Puncture wounds from a high-pressure injection may exhibit small entrance wounds, but the excessive force through the tissue planes, tendons, and associated structures may cause significant injury. A radiograph will display the radiopaque paint pattern. Warm saline soaks may increase swelling and ischemia. Direct application of ice may cause a cold injury to the finger.

Sheehy's Manual of Emergency Care, 6th ed., p. 206-207
Sheehy's Emergency Nursing, Principles and Practice, 5th ed., p. 146

 

Term

An elderly woman presents to the emergency department with vague complaints of abdominal pain. The patient states that the people she lives with are stealing from her. The emergency nurse should

 

A.

call the family to discuss the situation.

B.

arrange for protective custody of the patient.

C.

assess the patient for signs of neglect and maltreatment.

D.

obtain consent for evaluation by social services.

Definition

C.

assess the patient for signs of neglect and maltreatment.

Elder maltreatment cannot be assessed quickly or easily from a cluster of signs and vague presenting symptoms. Keen awareness when performing the physical examination is essential to identify elder abuse.

Sheehy's Emergency Nursing, Principles and Practice, 5th ed, p. 772
Emergency Nursing Core Curriculum, 6th ed., p. 60-62

Term

Which of the following will be found on a patient with a severe asthma exacerbation?

 

A.

prolonged inspiratory respirations

B.

expiratory or absent wheezing

C.

a loose and barking productive cough

D.

infiltrates on the chest radiograph

Definition

B.

expiratory or absent wheezing

Clinical manifestations include cough, wheezing, prolonged expiration time, and reduced peak expiratory flow rate. Increased work of breathing and use of accessory muscles may also be present.

Sheehy's Emergency Nursing, Principles and Practice, 5th ed., p. 431
Core Curriculum for Critical Care Nursing, 6th ed., 693-697
Sheehy's Manual of Emergency Care, 6th ed., p. 314-315

Term

A patient with complete A-V block is hypotensive and confused. The most appropriate treatment would be

 

A.

transcutaneous pacing.

B.

lidocaine (Xylocaine).

C.

defibrillation.

D.

carotid massage.

Definition

A.

transcutaneous pacing.

Transcutaneous pacing is the recommended immediate intervention for severely symptomatic patient in third degree (complete) heart block. The other therapies, Lidocaine, defibrillation and carotid massage would worsen this patient's condition.

American Heart Association - Advanced Cardiac Life Support Manual, 2006, p. 83

 

Term

Which of the following substances should be used to counteract the effects of heparin?

 

A.

vitamin K

B.

quinidine sulfate

C.

protamine sulfate

D.

sodium bicarbonate

Definition

C.

protamine sulfate

Protamine sulfate neutralizes heparin. With the presence of protamine, PTTs that are prolonged because of a heparin effect are reduced.

Nursing Drug Handbook, 2007, p. 1272-1273

 

Term

A potential organ donor has a signed donor card. Family consent is obtained prior to organ procurement in order to

 

A.

initiate the family's grieving process.

B.

avoid potential legal disputes with the family.

C.

inform the family of the need to delay the burial.

D.

offer the family the opportunity to direct the destination of the organs.

Definition

B.

avoid potential legal disputes with the family.

Consent for any invasive procedure, including organ donation, must be obtained to avoid legal disputes. Organ donation does not delay the burial process. Ultimate destination of the organs is determined by the organ procurement organization.

Emergency Nursing Core Curriculum, 6th ed., p. 131

 

Term

Reperfusion dysrhythmias following fibrinolytic therapy should be treated only when

 

A.

the patient reports cessation of chest pain.

B.

the area of infarct is believed to be the anterior wall.

C.

they cause physiological compromise to the patient.

D.

creatine kinase (CK/CPK) and CK-MB values are elevated.

Definition

C.

they cause physiological compromise to the patient.

When ischemic tissue is initially reperfused, it is acidotic and irritable. The stunned cells often function less than optimally until homeostasis is restored. Most reperfusion dysrhythmias following fibrinolytic therapy are of short duration and are well tolerated by patients. These dysrhythmias generally resolve spontaneously. Do not initiate treatment unless persistent dysrhythmias compromise the patient's hemodynamic status.

Sheehy's Manual of Emergency Care, 5th ed., p. 475

Term

A patient who has appendicitis is waiting to see a surgeon. The emergency nurse should prepare to administer

 

A.

IV colloids.

B.

IV crystalloids.

C.

oral antibiotics.

D.

oral analgesics.

Definition

B.

IV crystalloids.

The patient with an appendicitis is at an elevated risk for fluid volume deficit secondary to anorexia and vomiting; therefore, initiation of IV access for administration of crystalloid fluid to replace intravascular volume is the priority intervention. The patient would be NPO, so any ordered analgesics or antibiotics would be administered parenterally.

Emergency Nursing Core Curriculum, 6th ed., p. 176 and 560

 

Term

A woman has had lower abdominal pain and dysuria for the past 3 days. She has costovertebral tenderness on the right side, a temperature of 104º F (40º C), vomiting, and chills. The emergency nurse should prepare to

 

A.

administer IV antibiotics.

B.

administer IV analgesics.

C.

insert an indwelling urinary catheter.

D.

increase the patient's PO fluid intake.

Definition

A.

administer IV antibiotics.

Fever, dysuria, and costovertebral tenderness are classic symptoms of pyelonephritis. Pyelonephritis, in this patient, has progressed from a urinary tract infection. IV antibiotics will be required to treat this infection.

Emergency Nursing Core Curriculum, 6th ed., p. 396-397
Merck Manual

 

Term

A patient who sustained a leg laceration with a chain saw has not had tetanus immunization in over 10 years. The patient should receive

 

 

A.

0.5 mL of tetanus and diphtheria toxoids.

B.

0.5 mL of tetanus, diphtheria toxoids, and pertussis vaccine.

C.

250 U of tetanus immune globulin and 0.5 mL of tetanus and diphtheria toxoids.

D.

5 mL of immune (gamma) globulin.

Definition

C.

250 U of tetanus immune globulin and 0.5 mL of tetanus and diphtheria toxoids.

Tetanus toxoid (0.5 mL) provides active immunization, whereas tetanus immune globulin (250-500 U) provides passive immunization.

The Centers for Disease Control and Prevention advise that the tetanus vaccine also should contain diphtheria toxin. This combination (dT) is given a single 0.5 mL IM dose. If the wound is tetanus-prone, simultaneous administration of tetanus antitoxin, 250 U IM is recommended.

Sheehy's Manual of Emergency Care, 6th ed., p. 199.
Sheehy's Emergency Nursing, Principles and Practice, 5th ed., p. 153

Term

A patient who returns to the emergency department 3 days after a motor vehicle crash has pain in the left upper quadrant of the abdomen. Assessment reveals pain on palpation, and the patient is hemodynamically stable. The emergency nurse should be most concerned about the possibility of

 

A.

a pneumothorax.

B.

a costochondritis.

C.

a splenic hematoma.

D.

bleeding from a liver laceration.

Definition

C.

a splenic hematoma.

The spleen, located in the left upper abdominal quadrant, is the most commonly injured organ in patients with blunt abdominal trauma. Because the spleen (like the liver and kidneys) is an encapsulated organ, bleeding from the parenchyma is commonly well-contained within the capsule. Therefore, the patient remains hemodynamically stable and there is no free blood in the peritoneum. Most patients with a splenic subcapsular hematoma will heal nicely without intervention. However, the capsule can rupture, releasing blood into the abdominal cavity.

A pneumothorax is manifested by respiratory findings, costochondritis is characterized by pain with inspiration or chest palpation, and a liver laceration causes pain in the RIGHT upper abdominal quadrant.

Emergency Nursing Core Curriculum, 6th ed., p. 788

Term

An indwelling urinary catheter is ordered for a patient who has sustained a straddle injury to his external genitalia. Before catheter insertion, the emergency nurse should assess the patient for

 

A.

penile tenderness.

B.

scrotal or penile swelling.

C.

blood at the urinary meatus.

D.

the ability to void spontaneously.

Definition

C.

blood at the urinary meatus.

When assessing a trauma patient with blood at the urinary meatus, it is important to know that this is a sign of a possible pelvic fracture and that a urinary drainage catheter insertion is contraindicated.

Penile tenderness and scrotal or penile swelling are expected complaints after the described trauma, but alone are not a contraindication for a urinary drainage catheter insertion. The ability to void spontaneously is not an indication or contraindication for the insertion of a urinary drainage catheter.

Sheehy's Manual of Emergency Care, 6th ed., p. 707

Term

An adult with an estimated blood loss of 1 L should have volume replacement with

 

A.

3 liters of colloids.

B.

3 liters of crystalloids.

C.

3 units of blood.

D.

3 units of fresh-frozen plasma.

Definition

B.

3 liters of crystalloids.

While there is much research concerning the most beneficial volume replacement regimen, the current (2007) consensus and the current standard of care is to replace lost fluids with isotonic crystalloids such as normal saline or lactated Ringer's solution at a 3:1 ratio.

Sheehy's Emergency Nursing, Principles and Practice, 5th ed., 2003. p. 510
Sheehy's Manual of Emergency Care, 6th ed., p. 363

Term

Which of the following pieces of information would be useful in the assessment of a patient with suspected hepatitis A?

 

A.

color of stools

B.

time of last bowel movement

C.

history of diarrhea

D.

unusual odor of feces

Definition

A.

color of stools

With hepatitis A, stool becomes clay-colored due to loss of pigmentation from bile.

Tintinalli, Emergency Medicine, 6th ed., 2004, p. 567

 

Term

A 10-month-old infant who has a sudden onset of wheezing, cough, and stridor should immediately be assessed for

 

A.

presence of abdominal breathing.

B.

unequal breath sounds.

C.

axillary subcutaneous emphysema on palpation.

D.

a history of bronchodilator use.

Definition

B.

unequal breath sounds.

Sudden onset of respiratory distress frequently indicates aspiration of a foreign body, so assessment should include bilateral breath sounds to determine if there is obstruction.

ENPC, 3rd ed., p. 74

 

Term

After a tibial plateau fracture has been splinted, treated with ice, and elevated, the emergency nurse should be most concerned if the patient has

 

A.

increasing ecchymosis distal to the fracture.

B.

delayed capillary refill to the toes.

C.

pain in the affected leg.

D.

swelling at the fracture site.

Definition

B.

delayed capillary refill to the toes.

A tibial plateau fractures extends into the knee joint. Joint involvement may be complicated by neurovascular compromise and associated fractures. With any fracture or dislocation, neurovascular status should be assessed before and after immobilization. If neurovascular status is compromised, further reassessment and interventions are required.

Trauma Nursing Core Course Manual, 6th ed., p. 190
Core Curriculum for Critical Care Nursing, 6th ed., p. 927

 

Term

A patient with symptoms of fatigue, low-grade fever, and jaundice is diagnosed with hepatitis A. Which of the following statements indicates this patient has a good understanding of the discharge instructions?

 

A.

"I need to work on rebuilding my strength."

B.

"I need to avoid alcohol for 2 to 3 months."

C.

"I understand transmission is from my sexual partner."

D.

"I understand that my close contacts may need treatment."

Definition

D.

"I understand that my close contacts may need treatment."

Hepatitis A virus is transmitted by the oral-fecal route and is infectious 2 weeks before and 1 week after jaundice. Others who have been in close contact with the patient may need treatment. Transmission in this case may not have come from the sexual partner. Alcohol is contraindicated during the acute phase and rebuilding strength is not the best choice.

Emergency Nursing Core Curriculum, 6th ed., p. 452.

 

Term

A patient returns to the emergency department 1 week after being treated for a first-degree sprained ankle. The emergency nurse should expect to find

 

A.

decreased pain and edema.

B.

pain with weight bearing.

C.

bluish ecchymosis around the ankle.

D.

capillary refill time greater than 3 seconds.

Definition

A.

decreased pain and edema.

A first-degree ankle sprain produces slight pain, minimal swelling, and absent or minor ecchymosis. Therefore, it is reasonable to expect decreased pain and edema after 1 week of treatment.

Core Curriculum for Critical Care Nursing, 6th ed., p. 902

Term

A 14-year-old patient whose airway is obstructed is conscious and unable to speak. The emergency nurse should first

 

A.

remove the obstruction with Magill forceps.

B.

administer back blows.

C.

administer abdominal thrusts.

D.

encourage the patient to cough.

Definition

C.

administer abdominal thrusts.

The American Heart Association recommends performing abdominal thrusts for a conscious choking patient; this should generate enough pressure to force the foreign body out of the trachea. Back blows are reserved for infant choking victims, and, without proper visualization, the Magill forceps are likely to force the blockage deeper. If the patient is obstructed, there will not be an effective cough.

American Heart Association Emergency Cardiac Care Guidelines, 2005

Term

A patient with a dystonic reaction to a phenothiazide is successfully treated with diphenhydramine (Benadryl). The nurse should anticipate that the diphenhydramine will be

 

A.

gradually increased.

B.

discontinued immediately.

C.

prescribed for the next 48 hours.

D.

titrated according to the patient's response.

Definition

C.

prescribed for the next 48 hours.

Treatment of dystonic reactions with Benadryl quickly reverses the symptoms. Benadryl must be prescribed for 72 hours to prevent a recurrence.

The Harriet Lane Handbook, 16th ed., p. 46

 

Term

A patient has just been prescribed the "morning after pill," and the emergency nurse has administered the first dose. Which of the following statements indicates the patient understands the discharge teaching?

 

A.

"My menstrual period will occur in 3 to 4 days."

B.

"Using condoms will prevent me from getting pregnant."

C.

"The medication may make me feel nauseated."

D.

"I will take the other two tablets in 24 hours."

Definition

C.

"The medication may make me feel nauseated."

The "morning-after pill" is an estrogen and progesterone birth-control pill and causes nausea and vomiting. It is common for the treating health care provider to also prescribe an antiemetic. This treatment will not stimulate a menstrual cycle. Using condoms after the fact will not prevent a pregnancy. The usual course of dosing is 2 pills now and 2 more in 12 hours.

Sheehy's Manual of Emergency Care, 6th ed., p. 822 and 916

 

Term

Nitroglycerin reduces myocardial oxygen consumption because its primary effect is to

 

A.

reduce preload.

B.

increase afterload.

C.

slow the heart rate.

D.

enhance contractility.

Definition

A.

reduce preload.

By promoting vasodilation, nitroglycerin reduces blood return to the heart. This reduction in preload means that the heart does not have to work so hard. When the heart's workload is decreased, myocardial oxygen consumption drops and cardiac output usually improves.

Emergency Nursing Core Curriculum, 6th ed., p. 206

 

Term

The priority nursing intervention for a patient who has frostbite is to

 

A.

administer ordered pain medication.

B.

use friction massage to rapidly warm the affected extremity.

C.

maintain moistened dressings on the affected area.

D.

prepare for a STAT escharotomy.

Definition

A.

administer ordered pain medication.

Frostbite is very painful during re-warming, so pain management is a priority. Friction massage will further damage tissue and moistened dressings can further decrease the temperature of the tissue secondary to evaporational cooling. Escharotomy is rarely used, except in the extreme case of extensive tissue damage.

Sheehy's Manual of Emergency Care, 6th ed., p. 499-501
Core Curriculum for Critical Care Nursing, 6th Ed., p. 320-322

 

Term

An emergency nurse is splashed in the eye with the blood from a trauma patient who is HIV seropositive. The nurse should

 

A.

irrigate the eye with zidovudine (AZT).

B.

irrigate the eye with diluted hypochlorite solution.

C.

begin a regimen of Interferon.

D.

undergo serum testing for HIV.

Definition

D.

undergo serum testing for HIV.

Following direct exposure of healthcare workers to HIV infected blood, a baseline HIV test is indicated.

Healthcare workers who have had significant exposure to known HIV should undergo baseline blood testing. Medications such as mono-antiretroviral or combination antiretroviral therapy may also be started, but not Interferon. The eye would be flushed with normal saline or lactated Ringers, but not with AZT or hypochlorite solution.

Emergency Nursing Core Curriculum, 6th ed., p. 446

Term

Which of the following catheters is best suited for an adult who requires rapid infusion of large volumes of IV crystalloids?

 

A.

14-gauge, 5 in (12.5 cm)

B.

14-gauge, 1 in (2.5 cm)

C.

18-gauge, 5 in (12.5 cm)

D.

18-gauge, 1 in (2.5 cm)

Definition

B.

14-gauge, 1 in (2.5 cm)

The fluid administration rate is limited by the diameter and length of the tubing and catheter. The rule of thumb is that fluid will be delivered most rapidly when larger bore and shorter catheters are used.

Emergency Nursing Core Curriculum, 6th ed., p. 104

 

Term

A local anesthetic of lidocaine (Xylocaine) with epinephrine (Adrenalin) is contraindicated in the treatment of

 

A.

scalp wounds.

B.

animal bites.

C.

finger lacerations.

D.

eyebrow lacerations.

Definition

C.

finger lacerations.

Because epinephrine is a potent vasoconstrictor, it is never used in fingers, nose, penis, or toes. Epinephrine should only be used in highly vascular areas.

Sheehy's Manual of Emergency Care, 6th ed., p. 200
Sheehy's Emergency Nursing, Principles and Practice, 5th ed., p. 148-149

 

Term

What breath sounds would most likely be heard in a patient who has acute pulmonary edema?

 

A.

stridor

B.

pleuritic rub

C.

bilateral crackles (rales)

D.

diminished breath sounds

Definition

C.

bilateral crackles (rales)

Stridor is a sign of upper airway obstruction. Increased fluids in the lungs causes crackles, wheezes, and productive cough with frothy white sputum that may have a pink tinge to it.

Sheehy's Emergency Nursing, Principles and Practice, 5th ed., p. 444.
Sheehy's Manual of Emergency Care, 6th ed., p. 313-314
Core Curriculum for Critical Care Nursing, 6th ed., 9. 203-207

 

Term

A patient is a potential qualified organ donor if they

 

A.

have not required vasopressors.

B.

are less than 55 years old.

C.

have been maintained in a mild hypothermic state.

 

D.

are kept normothermic until donation is completed.

Definition

D.

are kept normothermic until donation is completed.

Patients maybe organ donors even if they have required vasopressors. Donor age range depends on what is to be donated. A 55-year-old can be a donor. Hypothermia can cause tissue damage and is not recommended for potential donor.

Sheehy's Emergency Nursing, Principles and Practice, 5th ed., p. 169-182
Emergency Nursing Core Curriculum, 6th ed., p. 134

Term

An infant dies suddenly in the emergency department. Which of the following would best assist the family members in the grieving process?

 

A.

Offer to provide a lock of the infant's hair.

B.

Provide assurance that things will be all right.

C.

Obtain prescriptions for sedatives.

D.

Give them a copy of the medical record.

Definition

A.

Offer to provide a lock of the infant's hair.

In unexpected infant death, there is no warning or way to prepare. By providing a lock of hair or other mementos, the emergency nurse will give the parents something by which to remember the child as they go through the grieving process.

Sheehy's Emergency Nursing Principles and Practice, 5th ed., p. 186
Sheehy's Manual of Emergency Care, 6th ed., p. 252

 

Term

A trauma patient complains of shoulder pain, but has no history or evidence of shoulder injury. The patient should be examined for possible

 

A.

pneumothorax.

B.

visceral injury.

C.

damage to the ulnar nerve.

D.

injury to the cervical spine.

Definition

B.

visceral injury.

Injury to the spleen can manifest itself with pain that is referred to the left shoulder (Kehr's sign). A visceral injury may be the cause of the referred pain, secondary to stimulation of the phrenic nerve due to abdominal distention or irritation of the diaphragm.

Trauma Nursing Core Course Manual, 6th ed., p. 154
Emergency Nursing Core Curriculum, 6th ed., p. 788

 

Term

A trauma patient has received multiple transfusions in the emergency department and starts to ooze blood from the IV sites, suggesting the development of

 

A.

sepsis.

B.

hemophilia.

C.

multisystem organ failure.

D.

disseminated intravascular coagulation.

Definition

D.

disseminated intravascular coagulation.

DIC involves simultaneous clotting and bleeding and is associated with various diagnoses, including trauma and multiple transfusions of banked blood. The patient may have acute bleeding or gradual blood loss. DIC is characterized by a prolonged PTT and oozing from a wound or puncture sites.

Core Curriculum for Critical Care Nursing, 6th ed., p. 414
Sheehy's Manual of Emergency Care, 6th ed., p. 121
Sheehy's Emergency Nursing Principles and Practice, 5th ed., p. 639

 

Term

Signs of a severe ankle sprain include

 

A.

nerve deficits distal to the injury.

B.

discoloration at the site of the injury.

C.

obvious angulation of the extremity.

D.

external rotation of the extremity.

Definition

B.

discoloration at the site of the injury.

A sprain is the stretching, separation, or tear of a supporting ligament. Sprains range from mild to severe and generally cause pain, swelling, and ecchymosis.

Options A, C, and D are incorrect because these findings suggest injury that involves bone, nerve, or joint tissue.

Emergency Nursing Core Curriculum, 6th ed. p. 901

Term

Which of the following is the best description of bronchiolitis?

 

A.

a bacterial infection with a sudden onset accompanied by decreased breath sounds

B.

a bacterial infection with a sudden onset resulting in inflammation of the epiglottis

C.

a viral infection with a slow onset causing lower respiratory tract inflammation

D.

a viral infection with a rapid onset accompanied by barking cough

Definition

C.

a viral infection with a slow onset causing lower respiratory tract inflammation

Bronchiolitis is a viral infection most commonly caused by respiratory syncytial virus, adenoviruses, and influenza. It characteristically affects the respiratory tract causing an inflammatory response that results in increased respiratory secretions. These secretions may cause obstruction in the smaller airways. Symptoms become progressively worse over a period of several days.

Emergency Nursing Core Curriculum, 6th ed., p. 698-699
ENPC, 3rd ed., p. 78

 

Term

Classic symptoms of Guillain-Barré syndrome include

 

A.

Kussmaul respirations.

B.

ascending weakness.

C.

descending paralysis.

D.

hyperactive reflexes.

Definition

B.

ascending weakness.

Classic symptoms of Guillain-Barré syndrome include ascending weakness or paralysis, paresthesia, and diminished or absent reflexes.

Emergency Nursing Core Curriculum, 6th ed., p. 532

Term

A manic episode of bipolar disorder is characterized by

 

A.

visual hallucinations.

B.

confusion.

C.

concrete thinking.

D.

euphoria.

Definition

D.

euphoria.

Signs and symptoms associated with manic episodes include elation or increased mental excitement, a demanding or euphoric manner, an increase in motor activity, irritable, and grandiose auditory hallucinations.

Sheehy's Manual of Emergency Care, 6th ed., p. 884
Core Curriculum for Critical Care Nursing, 6th ed., p. 677

 

Term

A nurse's primary goal in emergency department triage is

 

A.

the rapid admission of patients to the treatment area.

B.

to develop a plan of care for each patient seen.

C.

the immediate and brief assessment to determine acuity level.

D.

to assist with rapid registration to expedite electronic documentation.

Definition

C.

the immediate and brief assessment to determine acuity level.

During triage, the nurse's primary role is the assessment to determine acuity level. Once that is done, the sorting, rapid admission, and physician consultation can occur in a manner which matches the patient's needs to available resources.

Sheehy's Manual of Emergency Care, 6th ed., p. 68-69
Core Curriculum for Critical Care Nursing, 6th ed., p. 29

Term

A 175 lb (80 kg) adult patient who sustained electrical burns is administered an IV solution of lactated Ringer's containing sodium bicarbonate. Fluid administration is deemed adequate if the patient's

 

A.

serum pH is 7.25.

B.

urine output is 75 to 100 mL/h.

C.

cardiac monitor shows only rare unifocal PVCs.

D.

urine is pink tinged.

Definition

B.

urine output is 75 to 100 mL/h.

It is important to closely monitor urine output on all burn patients. This is especially true in the electrical burn patient, as the full extent of the burn injury many not be known. Urine output should be maintained (greater and equal to) 1 ml/kg/hr. The normal serum pH is 7.35-7.45. Rare unifocal PVCs occur even in non-burn patients.

Sheehy's Manual of Emergency Care, 6th ed., p. 772
Core Curriculum for Critical Care Nursing, 6th ed., p. 807, 816-818
Trauma Nursing Core Course Manual, 6th ed., p. 214

Term

Pulseless electrical activity may be identified clinically by

 

A.

a 12-lead ECG and heart sounds.

B.

ECG monitoring and assessment of the patient's pulse.

C.

the presence of heart sounds and distended neck veins.

D.

the measurement of blood pressure in both arms and heart rate.

Definition

B.

ECG monitoring and assessment of the patient's pulse.

Pulseless electrical activity is ALWAYS a clinical diagnosis, versus an ECG diagnosis. The only way to determine that a rhythm is pulseless is to check for pulsations/cardiac contraction. This can be done by palpating a pulse, listening with a stethoscope or Doppler, or visualizing the heart with ultrasound.

Emergency Nursing Core Curriculum, 6th ed., p. 232

Term

Ludwig's angina is suspected in a patient who has radiating neck pain. The emergency nurse should assess for

 

A.

expiratory wheezing.

B.

cardiac dysrhythmias.

C.

posterior cervical adenopathy.

D.

cellulitis of the floor of the mouth.

Definition

D.

cellulitis of the floor of the mouth.

Ludwig's angina is a bacterial cellulitis of the floor of the mouth/neck and NOT a cardiac condition. Posterior cervical adenopathy would not be a specific symptom of Ludwig's angina.

Sheehy's Manual of Emergency Care, 6th ed., p. 592
Emergency Nursing Core Curriculum, 6th ed., p. 259

Term

A patient who is trembling, sweating, and hyperventilating is having an anxiety attack. Arterial blood gas analysis is likely to show which of the following?

 

A.

pH < 7.35

B.

HCO3- level > 26 mEq/L

C.

oxygen saturation < 96%

D.

PaCO2 < 35 mm Hg

Definition

D.

PaCO2 < 35 mm Hg

With the increased respiratory rate that occurs with anxiety, carbon dioxide is blown off, which results in a low PaCO2.

Emergency Nursing Core Curriculum, 6th ed., p. 707

 

Term

An 82-year-old patient sustained a fractured hip in a recent fall. In evaluating the patient's condition, the emergency nurse should be especially concerned about the

 

A.

cause of the fall.

B.

time the fall occurred.

C.

degree that the affected leg is rotated outward.

D.

patient's history of drug allergies.

Definition

A.

cause of the fall.

Elderly patients are at risk for falls. The etiology of the fall is investigated to rule out a cardiac, vascular, or neurologic origin. The other options are components of the nursing history and assessment. However, the most important assessment at this time is to determine the etiology of the fall.

Sheehy's Manual of Emergency Care, 6th ed., p. 793-794
Core Curriculum for Critical Care Nursing, 6th ed., p. 908

 

Term

A patient complains of chest pain that worsens with inspiration or lying supine and lessens by leaning forward. The nurse should suspect

 

A.

costochondritis.

B.

pleurisy.

C.

endocarditis.

D.

pericarditis.

Definition

D.

pericarditis.

This is a classic description of pericarditis pain. Costochondritis worsens with manual pressure against the anterior chest wall. Pleuritic pain is aggravated by deep inspiration. Endocarditis is generally painless but is often associated with systemic illness.

Emergency Nursing Core Curriculum, 6th ed., p. 211-213

 

Term

A man who weighs 165 lb (75 kg) arrives at the emergency department immediately after sustaining a 40% partial-thickness burn. Using the formula 4 mL/kg multiplied by the percent of body surface area burned, IV fluids should be administered at what rate during the first 8 hours?

 

A.

  375 mL/h

B.

  500 mL/h

C.

  750 mL/h

D.

1000 mL/h

Definition

C.

  750 mL/h

Use the Parkland formula: 4 mL/kg/total body surface area of the burn.
The first half of the calculated amount of IV fluid should be given during the first 8 hours and the remainder should be given over the next 16 hours.
4 mL x 75 kg x 40% = 12,000/2 = 6,000/8 = 750 mL/h

Sheehy's Manual of Emergency Care, 6th ed., p. 769

Term

The earliest indication of increased intracranial pressure is

 

A.

abnormal respiration.

B.

decreased heart rate.

C.

widened pulse pressure.

 

D.

altered level of consciousness.

Definition

D.

altered level of consciousness.

Early signs and symptoms of increased intracranial pressure are changes in the level of consciousness and restlessness. Vital sign changes are a late sign of increased intracranial pressure.

Emergency Nursing Core Curriculum, 6th ed., p. 826

Term

Assessment of a patient who has acute cholecystitis is most likely to reveal

 

A.

epigastric pain that radiates to the back, and watery stools.

B.

pain in the left upper quadrant of the abdomen, and positive results on a guaiac test of vomitus.

C.

pain in the right upper quadrant of the abdomen, and slight scleral icterus.

D.

pain that radiates to the left shoulder, and positive results on a stool guaiac test.

Definition

C.

pain in the right upper quadrant of the abdomen, and slight scleral icterus.

Pain is caused by inflammation of the gall bladder and obstruction of the cystic duct from a gallstone. The icterus is caused by excess circulating bilirubin secondary to backup from common duct obstruction.

Sheehy's Manual of Emergency Care, 6th ed., p. 401
Merck Manual

 

Term

A patient becomes increasingly agitated after receiving 2 mg lorazepam (Ativan). The emergency nurse should recognize that this response is

 

A.

expected, since this is a typical side effect.

B.

expected, and the patient needs to be restrained.

C.

unexpected, and the patient needs more haloperidol.

D.

unexpected, since the condition is exacerbated.

Definition

D.

unexpected, since the condition is exacerbated.

Lorazepam (Ativan) is used to control acute delirium and agitation, especially in emergency situations. It is classified as an antipsychotic, sedative/tranquilizer. Agitation is not a common side effect of lorazepam, and restraints should only be used when there is risk of injury to the patient or others.

Sheehy's Manual of Emergency Care, 6th ed., p. 877

 

Term

Testicular torsion is usually associated with scrotal pain that has a

 

A.

sudden onset associated with nausea.

B.

gradual onset and an associated fever.

C.

sudden onset that resolves after urination.

D.

gradual onset that radiates into the lower abdomen.

Definition

A.

sudden onset associated with nausea.

A sudden onset of scrotal pain results from twisting of the spermatic cord. This results in cessation of blood flow and, ultimately, tissue necrosis if not diagnosed and treated rapidly.

Sheehy's Manual of Emergency Care, 6th ed., p. 518-519
Mosby's Emergency Nursing Reference, 3rd ed., p. 470

 

Term

An elderly patient presents to the emergency department with numbness and tingling of the right arm and right leg and expressive aphasia. Which of the following is the most important information to obtain?

 

A.

history of atrial fibrillation

B.

time of onset of symptoms

C.

recent use of aspirin products

D.

normal BP for patient

Definition

B.

time of onset of symptoms

There is a very narrow window of opportunity for use of fibrinolytic therapy and it must be started within 3 hours of symptom onset.

Sheehy's Emergency Nursing, Principles and Practice, 5th ed., 2003, p. 524

Term

Which of the following arterial blood gas measurements is consistent with diabetic ketoacidosis?

 

A.

pH = 7.26; PaCO2 = 40 mm Hg; PaO2 = 80 mm Hg

B.

pH = 7.34; PaCO2 = 32 mm Hg; PaO2 = 95 mm Hg

C.

pH = 7.24; PaCO2 = 25 mm Hg; PaO2 = 105 mm Hg

D.

pH = 7.48; PaCO2 = 35 mm Hg; PaO2 = 130 mm Hg

Definition

C.

pH = 7.24; PaCO2 = 25 mm Hg; PaO2 = 105 mm Hg

In DKA, the serum arterial or venous pH is less than 7.30. Blood gas analysis indicates a normal PaO2.

Core Curriculum for Critical Care Nursing, 6th ed., p. 298
Sheehy's Emergency Nursing, Principles and Practice, 5th ed., 2003 p. 602

 

Term

Upon attempting mouth-to-mask resuscitation, an emergency nurse finds resistance. The nurse should immediately

 

A.

roll the patient to one side and clean out the mouth.

B.

roll the patient to one side and deliver four quick blows to the back.

C.

turn the patient's head to one side and clean out the mouth.

 

D.

reposition the patient's airway and attempt ventilation again.

Definition

D.

reposition the patient's airway and attempt ventilation again.

AHA Guidelines in Basic Life Support state to reposition the head to ensure the tongue is not obstructing the airway and causing an obstruction.

American Heart Association-Advanced Cardiac Life Support Manual, 2006

Supporting users have an ad free experience!